スピントロニクス理論の基礎/8-1 のバックアップの現在との差分(No.5)

更新


  • 追加された行はこの色です。
  • 削除された行はこの色です。
[[スピントロニクス理論の基礎]]
~>>>> [[[次の章へ]>スピントロニクス理論の基礎/8-2]]

培風館 多々良源 [[「スピントロニクス理論の基礎」>http://www.amazon.co.jp/%E3%82%B9%E3%83%94%E3%83%B3%E3%83%88%E3%83%AD%E3%83%8B%E3%82%AF%E3%82%B9%E7%90%86%E8%AB%96%E3%81%AE%E5%9F%BA%E7%A4%8E-%E6%96%B0%E7%89%A9%E7%90%86%E5%AD%A6%E3%82%B7%E3%83%AA%E3%83%BC%E3%82%BA-%E5%A4%9A%E3%80%85%E8%89%AF-%E6%BA%90/dp/4563024406]] を勉強していく上で、気付いた誤植や難しい部分の注釈を書いておこうと思います。
* 8-1 物理量 [#h3d06217]

他の節についてはこちら > [[スピントロニクス理論の基礎]]
#contents

* 8-1 物理量 [#h3d06217]
** 量子統計平均 [#ic720f0c]

(8.1)

&math(&\overline O(t_0) \equiv \frac{1}{Z_0} \trace[e^{-\beta H(t_0)}O(t_0)]\\
&=\frac{1}{Z_0} \sum_{\alpha(t_0)} \braket{\alpha(t_0)|e^{-\beta H(t_0)}O(t_0)|\alpha(t_0)}\\
&\equiv \bigl\langle e^{-\beta H(t_0)}O(t_0) \bigr\rangle );~
&\equiv \bigl\langle \frac{e^{-\beta H(t_0)}}{\red{Z_0}}O(t_0) \bigr\rangle );~
&math(\equiv \bigllangle O(t_0) \bigrrangle);

&math(\langle \ \ \rangle); は &math(e^{-\beta H(t_0)}); を含まない~
&math(\llangle \ \  \rrangle); は &math(e^{-\beta H(t_0)}); を含む和になっている。
&math(\langle \ \ \rangle); は &math(e^{-\beta H(t_0)}\red{/Z_0}); を含まず~
&math(\llangle \ \  \rrangle); は &math(e^{-\beta H(t_0)}\red{/Z_0}); を含む和になっている。

できる限り &math((t_0)); をあらわに書いてみた。

&math(Z_0 \equiv \trace\left[e^{-\beta H(t_0)}\right]); は分配関数。

この Σ の部分を trace と呼んでいるのは、
&math(\ket{\alpha}); を基底として &math(O); を行列に表した際の
&math((\alpha,\alpha')); 要素が &math(\braket{\alpha|O|\alpha'}); であり、
Σ の部分はちょうど対角要素の和になっているためである。

線形代数で学ぶように、trace は &math(\ket{a}=U\ket{\alpha}); のような
ユニタリ変換による基底の付け替えに対して値が変化しない量であり、
基底の選び方に依らず値の決まる物理量である。


&math(\ket{\alpha}); がエネルギー固有関数の時は、

(8.2)

&math(H(t_0)\ket{\alpha(t_0)}=E_\alpha\ket{\alpha(t_0)});

&math(E_\alpha); は &math(\alpha); が &math(t=t_0); で持つエネルギーとして定義される。

このとき、

&math(
&\bra{\alpha(t_0)}e^{-\beta H(t_0)}
=\big[(e^{-\beta H(t_0)})^\dagger\ket{\alpha(t_0)}\big]^\dagger
=\big[e^{-\beta H^\dagger(t_0)}\ket{\alpha(t_0)}\big]^\dagger\\
&=\big[e^{-\beta H(t_0)}\ket{\alpha(t_0)}\big]^\dagger
=\big[e^{-\beta E_\alpha}\ket{\alpha(t_0)}\big]^\dagger
=e^{-\beta E_\alpha}\bra{\alpha(t_0)}
);

より、

(8.1A)

&math(&\overline O(t_0) =\frac{1}{Z_0} \sum_{\alpha(t_0)} e^{-\beta E_\alpha}\braket{\alpha(t_0)|O(t_0)|\alpha(t_0)});

trace を取ることが量子力学で期待値を求めることに他ならず(量子平均)~
そこに &math(e^{-\beta E_\alpha}); の因子を含めることで同時に統計平均も取れることになる。

&math(\ket{\alpha}); がエネルギーの固有関数でないときには、(8.1) 式は正しいのだろうか?
エネルギー固有関数でなければ、
エネルギー固有関数でなければ、一般に

&math(\frac{1}{Z_0} \trace[e^{-\beta H(t_0)}O(t_0)]\ne \frac{1}{Z_0} \trace[O(t_0)e^{-\beta H(t_0)}]);

となるが、どちらが目的にかなうかが明確ではない。
となるような気がするのだが、どちらが目的にかなうかが明確ではない。

恐らく、&math(\ket{\alpha}); はエネルギーの固有関数でなければ (8.1) 
は意味をなさないのではないか?言い換えると、(8.1) ではなく (8.1A) 
が基本式なのではないか、と思える。

(2015-02-20 追記) 上でも述べているとおり、トレースの値は任意のユニタリー変換に対して保存するから、異なる物理量に対する固有ベクトルで書き直してもトレースの値が変わることはない。(8.1) は基底の取り方によらず意味を持つ(のかもしれない・・・)。

** 時間発展演算子の積分表示 [#ied01102]

さて、&math(\ket{\alpha(t_0)}); はハミルトニアンの固有関数なので、
ハミルトニアンが時間的に変化しない限り、
&math(\ket{\alpha(t_0)}); も時間に対して位相以外変化しない。

(8.3)

&math(i\hbar\frac{\PD}{\PD t}\ket{\alpha(t)}=H(t)\ket{\alpha(t)});

&math(t>t_0); において &math(H(t)); が変化すると、この式に沿って量子状態も時間変化を始める。

(8.4)

&math(\ket{\alpha(t)}\equiv U(t,t_0)\ket{\alpha(t_0)});

このようにして定義した &math(U(t,t_0)); がユニタリ演算子となることが (8.10) で示される。

(8.5)

(8.3) より

&math(i\hbar\frac{\PD}{\PD t} U(t,t_0)=H(t) U(t,t_0));

(8.6)

両辺を積分して &math(U(t_0,t_0)=1); を使う。

&math(i\hbar \left[ U(t,t_0) \right]_{t_0}^t=\int_{t_0}^t dt_1 H(t_1) U(t_1,t_0));

&math(U(t,t_0)-U(t_0,t_0)=U(t,t_0)-1=-\frac{i}{\hbar}\int_{t_0}^t dt_1 H(t_1) U(t_1,t_0));

&math(U(t,t_0)=1-\frac{i}{\hbar}\int_{t_0}^t dt_1 H(t_1) U(t_1,t_0));

(8.7)

右辺の &math(U(t_1,t_0)); を分解していくと、

&math(&U(t,t_0)=1-\frac{i}{\hbar}\int_{t_0}^t dt_1 H(t_1) \left[
1-\frac{i}{\hbar}\int_{t_0}^{t_1} dt_2 H(t_2) U(t_2,t_0)
\right]\\
&=1+\frac{-i}{\hbar}\int_{t_0}^t dt_1 H(t_1)+\left(\frac{-i}{\hbar}\right)\int_{t_0}^t dt_1 \int_{t_0}^{t_1} dt_2 H(t_1)H(t_2) U(t_2,t_0)\\
&=1+\frac{-i}{\hbar}\int_{t_0}^t dt_1 H(t_1)
+\left(\frac{-i}{\hbar}\right)^2\int_{t_0}^t dt_1 \int_{t_0}^{t_1} dt_2 H(t_1)H(t_2)\\
&+\left(\frac{-i}{\hbar}\right)^3\int_{t_0}^t dt_1 \int_{t_0}^{t_1} dt_2 \int_{t_0}^{t_2} dt_3 H(t_1)H(t_2)H(t_3) + \cdots\\
&=\sum_{n=0}^\infty\left(\frac{-i}{\hbar}\right)^n\int_{t_0}^t dt_1 \int_{t_0}^{t_1} dt_2 \cdots \int_{t_0}^{t_{n-1}} dt_n H(t_1)H(t_2)\cdots H(t_n)\\
&\equiv Te^{\frac{-i}{\hbar}\int_{t_0}^tdt'H(t')}
);

この式から &math(T); が 「時間順序づけ演算子」 だと言われてもちょっとピンと来ない。
確かに積分の中では &math(H(t)); が時間の古い方が右側になるように並んでいるけれど・・・

この演算子について、この教科書では示されていないが、後に見る 
(8.7A) や (8.8A) の形で理解すべきな気がしているので、そちらも参照のこと。

** 逆時間発展演算子 [#q17566ed]

(8.9) は唐突なので、以下ちょっと話の流れを変える。

(8.11)

&math(&\left[U(t,t_0)\right]^\dagger\\
&=\sum_{n=0}^\infty\left(\frac{i}{\hbar}\right)^n\int_{t_0}^t dt_1 \int_{t_0}^{t_1} dt_2 \cdots \int_{t_0}^{t_{n-1}} dt_n H(t_n)H(t_{n-1})\cdots H(t_1)\\
&\equiv \overline Te^{\frac{-i}{\hbar}\int_{t_0}^tdt'H(t')});
&\equiv \overline Te^{\frac{i}{\hbar}\int_{t_0}^tdt'H(t')});

を考える。

&math((AB)^\dagger=B^\dagger A^\dagger); のように、エルミート共役により演算子のかけ算の順が入れ替わること、&math(H(t)); はエルミート演算子であること &math([H(t)]^\dagger=H(t));、エルミート共役で虚数単位の符号が反転すること &math(i\rightarrow -i); から、&math(U(t,t_0)); のエルミート共役が上記のように書けることが分かる。

これと (8.7) との積を作ってみると、

(8.9)

&math(&\left[U(t,t_0)\right]^\dagger U(t,t_0)\\
&=\overline Te^{\frac{i}{\hbar}\int_{\textcolor{red}{t_0}}^tdt'H(t')}\,
Te^{\frac{-i}{\hbar}\int_{\textcolor{red}{t_0}}^tdt'H(t')}\\
&=\left(1+\frac{i}{\hbar}\int_{t_0}^tdt_1H(t_1)+\left(\frac{i}{\hbar}\right)^2\int_{\textcolor{red}{t_0}}^t dt_1\int_{\textcolor{red}{t_0}}^{t_1}dt_2H(t_2)H(t_1)+\dots\right)\times\\
&\left(1+\frac{-i}{\hbar}\int_{t_0}^tdt_1H(t_1)+\left(\frac{-i}{\hbar}\right)^2\int_{\textcolor{red}{t_0}}^tdt_1\int_{\textcolor{red}{t_0}}^{t_1}dt_2H(t_1)H(t_2)+\dots\right)\\
&=1);

実際に最後の等号を示そうとすると大変であるが、ちゃんと計算すれば示せないことはない。

様子を見るため、&math((i/\hbar)); の2次の項を抜き出してみると、

(8.9A)

&math(
 &\int_{t_0}^tdt_1\int_{t_0}^{t_1}dt_2 H(t_2)H(t_1)
+\int_{t_0}^tdt_1\int_{t_0}^{t_1}dt_2 H(t_1)H(t_2)
-\int_{t_0}^tdt_1\int_{t_0}^{t}dt_2 H(t_1)H(t_2)\\
=&\int_{t_0}^tdt_2\int_{t_0}^{t_2}dt_1 H(t_1)H(t_2)
+\int_{t_0}^tdt_1\int_{t_0}^{t_1}dt_2 H(t_1)H(t_2)
-\int_{t_0}^tdt_1\int_{t_0}^{t}dt_2 H(t_1)H(t_2)\\
=&\int_{t_0}^tdt_2\int_{t_0}^tdt_1 \theta(t_2-t_1)H(t_1)H(t_2)
+\int_{t_0}^tdt_1\int_{t_0}^tdt_2 \theta(t_1-t_2)H(t_1)H(t_2)
-\int_{t_0}^tdt_1\int_{t_0}^{t}dt_2 H(t_1)H(t_2)\\
=&\int_{t_0}^tdt_2\int_{t_0}^tdt_1 [\theta(t_2-t_1)+\theta(t_1-t_2)-1]H(t_1)H(t_2)
= 0
);

となる。

-2行目では第1項の &math(t_1); と &math(t_2); の名前を付け替えた
-3行目では積分範囲を &math((t_0\rightarrow t)^2); に拡大し、
もとの範囲を &math(\theta); 関数で表した~
&math(\theta(t)=\begin{cases}0&(t<0)\\1&(0<t)\end{cases});
-4行目では &math(\theta(t)+\theta(-t)=1); を用いた

少し脱線するが、(8.7) に現れる Σ が指数関数の展開を表しているにも関わらず、
&math(\frac{1}{n!}); の因子が現れない理由が上記の &math(\theta); 関数の部分に
表された積分範囲にある。

これを見るために &math(H); が時間に依存しない場合を計算しておく。

(8.9B)

&math(
&\sum_{n=0}^\infty\left(\frac{-i}{\hbar}\right)^n\int_{t_0}^t dt_1 \int_{t_0}^{t_1} dt_2 \cdots \int_{t_0}^{t_{n-1}} dt_n H(t_1)H(t_2)\cdots H(t_n)\\
U(t,t_0)\\
&=\sum_{n=0}^\infty\left(\frac{-i}{\hbar}\right)^n\int_{t_0}^t dt_1 \int_{t_0}^{t_1} dt_2 \cdots \int_{t_0}^{t_{n-1}} dt_n H(t_1)H(t_2)\cdots H(t_n)\\
=&\sum_{n=0}^\infty\left(\frac{-i}{\hbar}\right)^n H^n \int_{t_0}^t dt_1 \int_{t_0}^{t_1} dt_2 \cdots \int_{t_0}^{t_{n-1}} dt_n 1\\
=&\sum_{n=0}^\infty\left(\frac{-i}{\hbar}\right)^n H^n \int_{t_0}^t dt_1 \int_{t_0}^{t_1} dt_2 \cdots \int_{t_0}^{t_{n-2}} dt_{n-1} (t_{n-1}-t_0)\\
=&\sum_{n=0}^\infty\left(\frac{-i}{\hbar}\right)^n H^n \int_{t_0}^t dt_1 \int_{t_0}^{t_1} dt_2 \cdots \int_{t_0}^{t_{n-3}} dt_{n-2} \left[\frac{1}{2}(t_{n-1}-t_0)^2\right]_{t_0}^{t_{n-2}}\\
=&\sum_{n=0}^\infty\left(\frac{-i}{\hbar}\right)^n H^n \int_{t_0}^t dt_1 \int_{t_0}^{t_1} dt_2 \cdots \int_{t_0}^{t_{n-4}} dt_{n-3} \left[\frac{1}{3!}(t_{n-2}-t_0)^3\right]_{t_0}^{t_{n-3}}\\
\vdots\\
=&\sum_{n=0}^\infty\left(\frac{-i}{\hbar}\right)^n H^n \left[\frac{1}{n!}(t_1-t_0)^n\right]_{t_0}^t\\
=&\sum_{n=0}^\infty\left(\frac{-i}{\hbar}\right)^n H^n \left[\frac{1}{n!}(t-t_0)^n\right]\\
=&\sum_{n=0}^\infty\frac{1}{n!}\left(\frac{-i}{\hbar} H (t-t_0)\right)^n\\
=&e^{-\frac{i}{\hbar}H\cdot(t-t_0)}
);

このように &math(n); 番目の項の積分範囲は &math((t-t_0)^n); の &math(1/n!); になっている。
また、この形から分かるように、&math(H); が時間に依らないときには &math(U); 自体が
&math(H); と可換になっている。

** 時間発展演算子の積表示 [#f4524bf5]

閑話休題で、(8.9) について:
&math(n>2); 次の項でも上と同様にゼロになることを示せるが、
この話は次のように考えることで、より直感的に理解できる。

(8.5) より &math(\frac{\PD}{\PD t}\Big[\log U(t,t_0)\Big]=\frac{-i}{\hbar}H(t));
以下、&math(t_2 \ge t_1 \ge t_0); として、

&math(\frac{\log U(t+\delta t, t_0) - \log U(t, t_0)}{\delta t} = \frac{-i}{\hbar}H(t));
(8.5) より &math(\frac{\PD\log U(t,t_1)}{\PD t}\bigg|_{t=t_2}=\frac{-i}{\hbar}H(t)\bigg|_{t=t_2});

&math(\log \frac{U(t+\delta t, t_0)}{U(t, t_0)} = \frac{-i}{\hbar}H(t)\delta t);
&math(\frac{\log U(t_2+\delta t, t_1) - \log U(t_2, t_1)}{\delta t} = \frac{-i}{\hbar}H(t_2));

&math(\log \frac{U(t_2+\delta t, t_1)}{U(t_2, t_1)} = \frac{-i}{\hbar}H(t_2)\delta t);

&math(\frac{U(t_2+\delta t, t_1)}{U(t_2, t_1)} = e^{\frac{-i}{\hbar}H(t_2)\delta t});

&math(U(t_2+\delta t, t_1) = e^{\frac{-i}{\hbar}H(t_2)\delta t}U(t_2, t_1));

したがって、

&math(U(t_0+\delta t, t_0) = e^{\frac{-i}{\hbar}H(t_0)\delta t}U(t_0, t_0) = e^{\frac{-i}{\hbar}H(t_0)\delta t});

&math(U(t_0+\delta t+\delta t', t_0) = e^{\frac{-i}{\hbar}H(t_0+\delta t)\delta t'}e^{\frac{-i}{\hbar}H(t_0)\delta t});
&math(U(t_0+\delta t+\delta t', t_0) = e^{\frac{-i}{\hbar}H(t_0+\delta t)\delta t'}U(t_0+\delta t,t_0) = e^{\frac{-i}{\hbar}H(t_0+\delta t)\delta t'}e^{\frac{-i}{\hbar}H(t_0)\delta t});

そこで時刻 &math(t_0); と &math(t); の間を &math(t>t_n>t_{n-1}>\cdots>t_2>t_1>t_0); に分割すれば、

(8.7A)

&math(U(t, t_0) = 
e^{\frac{-i}{\hbar}H(t_n)(t-t_n)}
e^{\frac{-i}{\hbar}H(t_{n-1})(t_n-t_{n-1})}
\cdots
\cdots 
e^{\frac{-i}{\hbar}H(t_1)(t_2-t_1)}
e^{\frac{-i}{\hbar}H(t_0)(t_1-t_0)}
); (8.7A)
);

一方で、
を得る。

一方で、この式のエルミート共役を取ってみる。これは個々の行列のエルミート共役を取り、
行列の積を入れ替え、&math(i); を &math(-i); に付け替えることで得られる。
ハミルトニアンがエルミート行列 &math(H(t_?)^\dagger=H(t_?)); 
であることを用いると、次式が得られる。

(8.8A)

&math([U(t, t_0)]^\dagger = 
e^{\frac{i}{\hbar}H(t_0)(t_1-t_0)}
e^{\frac{i}{\hbar}H(t_1)(t_2-t_1)}
\cdots
e^{\frac{i}{\hbar}H(t_{n-1})(t_n-t_{n-1})}
e^{\frac{i}{\hbar}H(t_n)(t-t_n)}); (8.8A)
e^{\frac{i}{\hbar}H(t_n)(t-t_n)});

(8.7) や (8.8) の意味をこのように理解していると物理的な理解が深まる。

これらを用いれば、

&math(&[U(t, t_0)]^\dagger U(t, t_0) =\\
&
e^{\frac{i}{\hbar}H(t_0)(t_1-t_0)}
e^{\frac{i}{\hbar}H(t_1)(t_2-t_1)}
\cdots
e^{\frac{i}{\hbar}H(t_{n-1})(t_n-t_{n-1})}
e^{\frac{i}{\hbar}H(t_n)(t-t_n)}\\
&e^{\frac{-i}{\hbar}H(t_n)(t-t_n)}
e^{\frac{-i}{\hbar}H(t_{n-1})(t_n-t_{n-1})}
\cdots
e^{\frac{-i}{\hbar}H(t_1)(t_2-t_1)}
e^{\frac{-i}{\hbar}H(t_0)(t_1-t_0)}\\
&=1); (8.9A)

は一目瞭然である。
は一目瞭然であるし、&math(U(t, t_0)); と途中の任意の時刻 &math(t_k); ただし 
&math(t>t_k>t_0); で分割して、

&math(
U(t, t_0) &= 
\underbrace{
e^{\frac{-i}{\hbar}H(t_n)(t-t_n)}
e^{\frac{-i}{\hbar}H(t_{n-1})(t_n-t_{n-1})}
\cdots 
e^{\frac{-i}{\hbar}H(t_{k})(t_{k+1}-t_{k})}
}_{U(t,t_k)}
\underbrace{
e^{\frac{-i}{\hbar}H(t_{k-1})(t_{k}-t_{k-1})}
\cdots 
e^{\frac{-i}{\hbar}H(t_1)(t_2-t_1)}
e^{\frac{-i}{\hbar}H(t_0)(t_1-t_0)}
}_{U(t_{k},t_0)}
\\&=
U(t,t_k)U(t_k,t_0)
);

と表せることも容易に理解できる。
(物理的には &math(t_0); から &math(t); までの時間発展が
&math(t_0); から &math(t_k); までの時間発展と
&math(t_k); から &math(t); までの時間発展と
を続けて行うことにより得られるのは自然であるが、
教科書の (8.7) の式を見ていてもなかなか理解できないのではないか)

注)これらの表記では &math(n\rightarrow \infty); として、
個々の &math(t_{k+1}-t_k); が無限小の時間間隔を表していると考えている。

(8.7A) の指数関数の積の部分を指数の和に、さらに積分に直すことで、

&math(U(t, t_0) &= 
e^{\frac{-i}{\hbar}H(t_n)(t-t_n)+\frac{-i}{\hbar}H(t_{n-1})(t_n-t_{n-1})+
\cdots +\frac{-i}{\hbar}H(t_1)(t_2-t_1)+\frac{-i}{\hbar}H(t_0)(t_1-t_0)}\\
&= e^{\frac{-i}{\hbar}[H(t_n)(t-t_n)+H(t_{n-1})(t_n-t_{n-1})+\cdots +H(t_1)(t_2-t_1)+H(t_0)(t_1-t_0)]}\\
&= e^{\frac{-i}{\hbar}[H(t_n)(t-t_n)+\sum_{k=1}^n H(t_{k-1})(t_k-t_{k-1})]}\\
&= e^{\frac{-i}{\hbar}T\int_{t_0}^t dt'H(t')}\\
&= Te^{\frac{-i}{\hbar}\int_{t_0}^t dt'H(t')}\\
);

のようにして (8.7) の最終式を理解できる。

この表記では、&math(T); や &math(\overline T); 
は &math(e^{\frac{i}{\hbar}H(t_k)(t_{k+1}-t_k)}); 
のような項をどちらから順に並べるかを表す演算子であると理解できる。
のような項をどちらから順に並べるかを表す演算子であると自然に理解できる。

(8.48) などで &math(T); が &math(H(t)); ばかりでなく &math(O(t)); 
も含めて並べ替える話になるが、このときには無数に並ぶ
&math(e^{\frac{i}{\hbar}H(t_k)(t_{k+1}-t_k)}); の間に
&math(O(t)); が挟まる形を思い浮かべれば良い。

(8.10)

上記より、&math([U(t,t_0)]^\dagger=U(t,t_0)^{-1}); であり、
&math(U(t,t_0)); がユニタリ演算子であることが証明された。

*** 疑問点 [#cd9807cc]

以下 2012/1/30 のセミナーで出た話の内容を追記:

&math(U(t,t_0)); がユニタリ演算子であることは、
シュレーディンガー方程式が時間反転に対して対称であることに由来している。

不純物散乱などを入れた結果、
系が時間反転に対して対称でなくなった場合におかしなことが起きないかどうか、
注意深く考えなければならない。

(2013-07-01 追記)→ ここで展開している理論では、
統計力学で言うところの微視的状態のそれぞれは時間反転に対して対称な、
純粋にシュレーディンガー方程式に沿って時間発展を行う波動関数であり、
量子統計平均後の物理量がその熱力学的な時間の矢に従って時間反転に対して非対称な時間発展を行う。

そのあたり、注意して読み進めるべき。

** 物理量の時間発展 [#y66befbc]

(8.12)

&math(\overline O(t)&=\frac{1}{Z_0}\sum_\alpha e^{-\beta E_\alpha} \braket{\alpha(t)|O(t)|\alpha(t)}\\
&=\frac{1}{Z_0}\textcolor{red}{\sum_\alpha} \braket{\alpha(t_0)|e^{-\beta H(t_0)}[U(t,t_0)]^\dagger O(t)U(t,t_0)|\alpha(t_0)}
);

この式は前半部分の最重要の式である。
特に、&math(e^{-\beta E_\alpha}); の部分に現れるエネルギーが時刻 &math(t); での値ではなく
&math(t_0); の時の値であることに注目したい。

&math(t=t_0); において系は熱平衡にあり、各エネルギー固有状態 &math(\ket{\alpha}); に &math(e^{-\beta E_\alpha}/Z_0); の確率で存在していた。量子力学的に見れば、それぞれの量子状態は系のハミルトニアンが変化しない限り安定で、お互いの間を行き来することはない。

その後ハミルトニアンが変化して系が熱平衡からずれると、
&math(t=t_0); における固有状態 &math(\ket{\alpha}); は (8.3) に従って時間発展する。
この時間発展が (8.4) のようにユニタリ変換で表せることから分かるとおり、
この時間発展はどんな &math(H(t)); に対してさえ決定論的であり、
また未来永劫、&math(U\ket{\alpha}); が
&math(t=t_0); における別の固有状態 &math(\ket{\alpha'}); 
が時間発展した &math(U\ket{\alpha'}); と混じり合うことはない。
これは &math(\braket{\alpha(t)|\alpha'(t)}=\braket{\alpha|U^\dagger U|\alpha'}
=\braket{\alpha|\alpha'}=\delta_{\alpha,\alpha'});
からも確かめられる。

したがって、時刻 &math(t); における物理量 &math(O); の統計平均は、
したがって、時刻 &math(t); における物理量 &math(O); の統計平均 &math(\overline O(t)); は、
時刻 &math(t); における状態 &math(\ket{\alpha(t)}); に対する &math(O(t)); の期待値
&math(\braket{\alpha|O(t)|\alpha}); に、&math(t=t_0); において系が状態
&math(\ket{\alpha}); にいた確率 &math(e^{-\beta E_\alpha}/Z_0); を掛けて、
&math(\braket{\alpha(t)|O(t)|\alpha(t)}); に、 &math(t=t_0); において系が状態 &math(\ket{\alpha}); にいた確率 &math(e^{-\beta E_\alpha}/Z_0); を掛けて、
(8.12) のようにして求められるのである。

再度、用いる確率分布が &math(t=t); ではなく &math(t=t_0); の物であることに注意せよ。

もちろん、系が時刻 &math(t); において熱平衡にある必要は無いし、
ハミルトニアンの変化がどれだけ急峻であったとしても (8.12) は成り立つ。
ここまでの議論には断熱近似等は入っていない(と私は理解しています)。

** 物理量演算子のハイゼンベルク表示 [#b03bf358]

(8.13)

&math(O_H(t)\equiv [U(t,t_0)]^\dagger O(t)U(t,t_0));

(8.14)

&math(\overline O(t)&=\frac{1}{Z_0}\sum_\alpha \braket{\alpha(t_0)|e^{-\beta H(t_0)}O_H(t)|\alpha(t_0)}\\
&=\frac{1}{Z_0}\sum_\alpha e^{-\beta E_\alpha} \braket{\alpha(t_0)|O_H(t)|\alpha(t_0)}
);

(8.15) 以下の議論は分かりにくい上に間違っている?

(8.17) は (8.13) そのものであるが、これを微分することにより、

(8.18)

&math(&\dot O_H \\
&= \dot U^\dagger O U\textcolor{red}{ + U^\dagger \dot O U} + U^\dagger O \dot U\\
&= \left[\frac{i}{\hbar}U^\dagger H\right] O U \textcolor{red}{+ \left(\frac{\PD O(t)}{\PD t}\right)_H} + U^\dagger O \left[\frac{-i}{\hbar}HU\right]\\
&=\frac{i}{\hbar}U^\dagger(HO-OH)U \textcolor{red}{+ \left(\frac{\PD O(t)}{\PD t}\right)_H}\\
&=\frac{i}{\hbar}(U^\dagger HU U^\dagger OU-U^\dagger OUU^\dagger HU) \textcolor{red}{+ \left(\frac{\PD O(t)}{\PD t}\right)_H}\\
&=\frac{i}{\hbar}(H_HO_H-O_HH_H) \textcolor{red}{+ \left(\frac{\PD O(t)}{\PD t}\right)_H}\\
&=\frac{i}{\hbar}\left[H_H(t), O_H(t)\right] \textcolor{red}{+ \left(\frac{\PD O(t)}{\PD t}\right)_H});

ここで、(8.5) およびそのエルミート共役である、

(8.5A)

&math(-i\hbar\frac{\PD}{\PD t}U(t,t_0)^\dagger=U(t,t_0)^\dagger H(t));

さらに、(8.9) で得た &math(U^\dagger U = U U^\dagger = 1); を使った。そして、

(8.16)

&math(H_H(t)=U(t,t_0)^\dagger H(t_0) U(t,t_0));

は、Heisenberg 表示でのハミルトニアンである。

ということで、

(8.15)

&math(&\dot O_H =\frac{i}{\hbar}\left[H_H(t), O_H(t)\right] \textcolor{red}{+ \left(\frac{\PD O(t)}{\PD t}\right)_H});

が正しいような・・・

ただしこの場合の &math(\frac{\PD O(t)}{\PD t}); は演算子が陽に時刻に依存していない場合 &math(O(t)=O); にはゼロになるため、次章で扱うように &math(O=c); であったり、&math(O=n); であったりする場合には &math(\frac{\PD O(t)}{\PD t}=0); で、結局 (8.15) と同じ形になります。
ただしこの場合の &math(\frac{\PD O(t)}{\PD t}); 
は演算子が陽に時刻に依存していない場合にはゼロになるため、次章で扱うように 
&math(O=c); であったり、&math(O=n); であったりする場合には &math(\frac{\PD O(t)}{\PD t}=0); 
で、結局 (8.15) と同じ形になる。

----

(8.16) の脚注:

&math(H(t) = H); のように時間に依存しない場合には、
&math(H(t) = H); のように時間に依存しない場合には、(8.9B) でも見たように

(8.7B)

&math(&Te^{\textcolor{red}{\frac{-i}{\hbar}}\int_{\textcolor{red}{t_0}}^{\textcolor{red}{t}}dt'H(t')}\\
&=e^{\textcolor{red}{\frac{-i}{\hbar}}H\int_{\textcolor{red}{t_0}}^{\textcolor{red}{t}}dt'}\\
&=e^{\textcolor{red}{\frac{-i}{\hbar}}H(t-t_0)}); (8.7B)
&=e^{\textcolor{red}{\frac{-i}{\hbar}}H(t-t_0)}); 

となる。

これは、(8.7A) からも

(8.7C)

&math(&U(t, t_0) = 
e^{\frac{-i}{\hbar}H(t_n)(t-t_n)}
e^{\frac{-i}{\hbar}H(t_{n-1})(t_n-t_{n-1})}
\cdots
e^{\frac{-i}{\hbar}H(t_1)(t_2-t_1)}
e^{\frac{-i}{\hbar}H(t_0)(t_1-t_0)}\\
&=e^{\frac{-i}{\hbar}H[(t-t_n)+(t_n-t_{n-1})+
\cdots
+(t_2-t_1)+(t_1-t_0)]}\\
&=e^{\frac{-i}{\hbar}H(t-t_0)}
); (8.7C)
); 

として確認できる。

* 質問・コメント [#oc9ef424]

#article_kcaptcha

// **私の勘違いでした。 [#uc0d15ea]
// >[[東北大CMPT M2]] (&timetag(2017-05-04T05:19:54+09:00, 2017-05-04 (木) 14:19:54);)~
// ~
// 下の質問は取り消させてください。~
// 私の勘違いでした。~
// http://park.itc.u-tokyo.ac.jp/kato-yusuke-lab/nagai/note_070106_in.pdf~
// を読んで納得しました。~
// 
// //
// 
// #comment_kcaptcha
// 
// **結局(8.7)は不適切な表現ですよね。。 [#v722a8e7]
// >[[東北大CMPT M2]] (&timetag(2017-05-04T05:10:17+09:00, 2017-05-04 (木) 14:10:17);)~
// ~
// (8.7)についてですが、結局、Hが時間発展してしまうとn!の逆数は現れないわけですから、Texp[\int^t_(t_0) dt' H(t')]と書くのは間違いだと思うのですが、どうでしょうか。~
// 
// //
// 
// #comment_kcaptcha


Counter: 8220 (from 2010/06/03), today: 3, yesterday: 0